ChaseDream

标题: PREP-1-75 sales of cigarettes [打印本页]

作者: hughlv    时间: 2010-4-5 21:33
标题: PREP-1-75 sales of cigarettes
75.(33198-!-item-!-188;#058&007254)



In the year following an eight-cent increase in the federal tax on a pack of cigarettes, sales of cigarettes fell ten percent.In contrast, in the year prior to the tax increase, sales had fallen one percent.The volume of cigarette sales is therefore strongly related to the after-tax price of a pack of cigarettes.

Which of the following, if true, would most strengthen the argument above?


A. During the second year after the tax increase, cigarette sales increased by a significant amount.

B. The information available to consumers on the health risks of smoking remained largely unchanged in the period before and after the tax increase.

C. Most consumers were unaware that the tax on cigarettes was going to increase.

D. During the year following the cigarette tax increase, many consumers had less income, in inflation-adjusted dollars, than they had had in the previous year.

E. During the year after the tax increase, there was a greater variety of cigarettes on the market than there had been during the previous year.

OA是B。我选了D,当时做的时候觉得不太对劲,但没理清楚思路为什么B才是正确答案。B只是排除了一个他因,但是能比D更加的strengthen呢?

在这个版我一般是帮忙回答问题,很少问,但这个确实不太明白。

作者: AmyGMAT    时间: 2010-4-5 21:55
我不是非常确定~但我觉得income减少了 并不一定意味着他们会减少购买~
作者: salasbond    时间: 2010-4-7 13:09
应该选B,题目问的是strengthen的选项,就是说要选出可以rule out 其他可能的选项。 B rules out the possibility for consumers to be wise to the health risk. 所以选b。
D是不对的。 D提出了一种新的可能,就是赚的少了。 所以D不是strengthen, but weakens the argument.
作者: dajiahaoh    时间: 2010-4-7 17:20
应该选B,题目问的是strengthen的选项,就是说要选出可以rule out 其他可能的选项。 B rules out the possibility for consumers to be wise to the health risk. 所以选b。
D是不对的。 D提出了一种新的可能,就是赚的少了。 所以D不是strengthen, but weakens the argument.
-- by 会员 salasbond (2010/4/7 13:09:13)



D应该也是加强了论证才对。且B说了健康信息,和税收导致销量下降有关吗?
作者: bonfin    时间: 2011-10-20 20:55
应该选B,题目问的是strengthen的选项,就是说要选出可以rule out 其他可能的选项。 B rules out the possibility for consumers to be wise to the health risk. 所以选b。
D是不对的。 D提出了一种新的可能,就是赚的少了。 所以D不是strengthen, but weakens the argument.
-- by 会员 salasbond (2010/4/7 13:09:13)

defender




欢迎光临 ChaseDream (https://forum.chasedream.com/) Powered by Discuz! X3.3